CBSE Questions for Class 11 Commerce Applied Mathematics Limits And Continuity     Quiz 8 - MCQExams.com

$$ \displaystyle \lim _{x \rightarrow 0} \dfrac{\cos (\tan x)-\cos x}{x^{4}} $$ is equal to
  • 1/6
  • -1/3
  • 1/2
  • 1
The value of $$ \displaystyle \lim _{n \rightarrow \infty}\left[\dfrac{1}{n}+\dfrac{e^{1 / n}}{n}+\dfrac{e^{2 / n}}{n}+\ldots .+\dfrac{e^{(n-1) / n}}{n}\right] $$ is
  • 1
  • 0
  • e-1
  • e+1
 $$\displaystyle \lim _{x \rightarrow 0} \dfrac{x^{4}\left(\cot ^{4} x-\cot ^{2} x+1\right)}{\left(\tan ^{4} x-\tan ^{2} x+1\right)} $$ is equal to
  • 1
  • 0
  • 2
  • None of these
$$ \displaystyle \lim _{n \rightarrow \infty} \sum_{x=1}^{20} \cos ^{2 n}(x-10) $$ is equal to
  • 0
  • 1
  • 19
  • 20
The value of $$\displaystyle \underset { n\rightarrow \infty  }{ lim } \left( \frac { 1 }{ n+1 } +\frac { 1 }{ n+2 } +...+\frac { 1 }{ 6n }  \right) $$ is
  • $$\displaystyle \log { 2 } $$
  • $$\displaystyle \log { 6 } $$
  • $$\displaystyle 1$$
  • $$\displaystyle \log { 3 } $$
A point where function $$f(x)$$ is not continuous where $$f(x)=\left[ \sin { \left[ x \right]  }  \right] $$ in $$\left( 0,2\pi  \right) $$; is ($$\left[ \ast  \right] $$ denotes greatest integer $$\le x$$)
  • $$(3,0)$$
  • $$(2,0)$$
  • $$(1,0)$$
  • $$(4,-1)$$
$$\underset { x\rightarrow \pi /4 }{ Lim } \dfrac { 2\sqrt { 2 } \left( cosx+sinx \right) ^{ 3 } }{ 1-sin2x } =2$$ is equal to
  • $$\dfrac{3\sqrt{2}}{2}$$
  • $$2\sqrt{2}$$
  • $$\dfrac{4\sqrt{2}}{3}$$
  • $does \not \exist$$
$$\displaystyle  \lim _{x \rightarrow 0} \dfrac{\sin \left(x^{2}\right)}{\ln \left(\cos \left(2 x^{2}-x\right)\right)} $$ is equal to
  • 2
  • -2
  • 1
  • -1
$$ \displaystyle \lim _{x \rightarrow 0} \dfrac{x \tan 2 x-2 x \tan x}{(1-\cos 2 x)^{2}} $$ is equal to
  • 2
  • -2
  • 1/2
  • -1/2
$$\displaystyle \lim _{x \rightarrow 0} \dfrac{1}{x}\left[\int_{y}^{a} e^{\sin ^{2} t} d t-\int_{x+y}^{a} e^{\sin ^{2} t} d t\right]$$ is equal to
  • $$e^{\sin ^{2} y}$$
  • $$\sin 2 y e^{\sin ^{2} y}$$
  • 0
  • None of these
If $$y^{r}=\dfrac{n !^{n+r-1} C_{r-1}}{r^{n}},$$ where $$n=k r(k \text { is constant }),$$ then $$\operatorname{lim}_{r\rightarrow\infty} y$$ is equal to
  • $$(k-1) \log _{e}(1+k)-k$$
  • $$(k+1) \log _{e}(k-1)+k$$
  • $$(k+1) \log _{e}(k-1)-k$$
  • $$(k-1) \log _{e}(k-1)+k$$
The value of $$ \displaystyle \lim _{x \rightarrow a} \sqrt{a^{2}-x^{2}} \cot \dfrac{\pi}{2} \sqrt{\dfrac{a-x}{a+x}} $$ is
  • $$\dfrac{2 a}{\pi} $$
  • $$-\dfrac{2 a}{\pi} $$
  • $$ \dfrac{4 a}{\pi} $$
  • $$-\dfrac{4 a}{\pi} $$
If function $$f(x)=\dfrac{x^2-9}{x-3}$$ is continuous at $$x=3$$, then value of $$(3)$$ will be:
  • $$6$$
  • $$3$$
  • $$1$$
  • $$0$$
If $$f(x)=\begin{cases} \begin{matrix} \dfrac{\log (1+mx)- \log (1-nx)}{x}; & x \ne 0 \end{matrix} \\ \begin{matrix} k; & x=0 \end{matrix} \\ \begin{matrix}  &  \end{matrix} \end{cases}$$
is continuous at $$x=0$$ then the value of $$k$$ will be:
  • $$0$$
  • $$m+n$$
  • $$m-n$$
  • $$m.n$$
If function $$f(x)=\begin{cases} \begin{matrix} \dfrac{\sin 3x}{x}; & x \ne 0 \end{matrix} \\ \begin{matrix} m; & x=0 \end{matrix} \\ \begin{matrix}  &  \end{matrix} \end{cases}$$
is continuous at $$x=2$$ then value of $$m$$ will be:
  • $$3$$
  • $$1/3$$
  • $$1$$
  • $$0$$
The value of $$\displaystyle \lim_{x\rightarrow 0} \dfrac {xe^{x} - \log_{e} (1 + x)}{x^{2}}$$ is
  • $$2/3$$
  • $$1/3$$
  • $$1/2$$
  • $$3/2$$
If $$f(x)=\begin{cases} \dfrac { 1-\sqrt { 2 } \sin { x }  }{ \pi -4x } ,\quad \quad ifx\neq \dfrac { \pi  }{ 4 }  \\ a\quad \quad \quad \quad \quad \quad \quad \quad \quad \quad ,\quad \quad ifx=\dfrac { \pi  }{ 4 }  \end{cases}$$ is continous at $$x=\dfrac {\pi}{4}$$ then $$a=$$
  • $$4$$
  • $$2$$
  • $$1$$
  • $$1/4$$
The value of $$\displaystyle \lim_{x\rightarrow \infty} \dfrac {\sin x}{x}$$ is
  • $$0$$
  • $$\infty$$
  • $$1$$
  • $$-1$$
The value of $$\displaystyle \lim_{x\rightarrow 0} \dfrac {1 - \cos x}{x^{2}}$$ is
  • $$0$$
  • $$1/2$$
  • $$-1/2$$
  • $$-1$$
The value of $$\displaystyle \lim_{x\rightarrow 0} \left (\dfrac {\sin 3x}{\tan x}\right )^{4}$$ is
  • $$0$$
  • $$81$$
  • $$4$$
  • $$1$$
The value of $$\displaystyle \lim_{x\rightarrow \infty} \sin \dfrac {\pi}{4x} \cos \dfrac {\pi}{4x}$$ is
  • $$\pi/4$$
  • $$\pi/2$$
  • $$0$$
  • $$\infty$$
Let $$f(x)=\displaystyle \frac{(256+ax)^{1/8}-2}{(32+bx)^{1/5}-2}$$. If $$f$$ is continuous at $$x = 0$$, then the value of $$a / b$$ is:
  • $$\displaystyle \frac{8}{5}f(0)$$
  • $$\displaystyle \frac{32}{5}f(0)$$
  • $$\displaystyle \frac{64}{5}f(0)$$
  • $$\displaystyle \frac{16}{5}f(0)$$
Let $$\alpha(a)$$ and $$\beta(a)$$ be the roots of the equation $$(\sqrt[3]{1+a}-1)x^{2}+(\sqrt{1+a}-1){x}+(\sqrt[6]{1+a}-1)=0$$ where $$a>-1$$. Then $$ \underset{a\rightarrow 0^{+}}{\lim}\alpha(a)$$ and $$ \underset{a\rightarrow 0^{+}}{\lim}\beta(a)$$ are 
  • $$-\displaystyle \frac{5}{2}$$ and 1
  • $$-\displaystyle \frac{1}{2}$$ and $$-1$$
  • $$-\displaystyle \frac{7}{2}$$ and 2
  • $$-\displaystyle \frac{9}{2}$$ and 3
If $$f(x)=\left\{\begin{matrix}(cos x)^{1/sinx} &for &x\neq 0 \\  k & for & x=0 \end{matrix}\right.$$
Then the value of $$k$$, so that $$f$$ is continuous at $$x=0$$ is
  • $$0$$
  • $$1$$
  • $$\dfrac{1}{2}$$
  • $$2$$
The function $$f$$ : $$R/\{0\}\rightarrow R$$ given by $$f(x)=\displaystyle \frac{1}{x}-\frac{2}{e^{2x}-1}$$ can be made continuous at $$x=0$$ by defining $$f(0)$$ as -
  • $$2$$
  • $$-1$$
  • $$0$$
  • $$1$$
$$\displaystyle f\left( x \right)=\begin{cases} \begin{matrix} \frac { \cos ^{ 2 }{ x } -\sin ^{ 2 }{ x } -1 }{ \sqrt { { x }^{ 2 }+4 } -2} , & x\neq 0 \end{matrix} \\ \begin{matrix} a, & x=0 \end{matrix} \end{cases}$$ then the value of $$a$$ in order that $$f(x)$$ may be continuous at $$x=0$$ is
  • $$-8$$
  • $$8$$
  • $$-4$$
  • $$4$$
If $$\displaystyle f(x)=\frac{\sin 3x+A\sin 2x+B\sin x}{x^{5}};x\neq 0$$ is continuous at $$x=0$$ , then
  • $$\displaystyle {A}+{B}=2$$
  • $$\displaystyle {A}+{B}=1$$
  • $$\displaystyle {A}+{B}=0$$
  • $$\displaystyle {A}-{B} =1$$
$$f(x)=\left\{\begin{array}{ll}\dfrac{(x+bx^{2})^{1/_{2}}-x^{1/2}}{bx^{3/2}} & x>0\\c & x=0\\\dfrac{\sin(a+1)x+\sin x}{x} & x<0\end{array}\right.$$ is continuous at $${x}=0$$, then
  • $$a=\displaystyle \frac{-3}{2},b=0,c=\frac{1}{2}$$
  • $$a=\displaystyle \frac{-3}{2},b\neq 0,c=\frac{1}{2}$$
  • $$a=\displaystyle \frac{3}{2},b\neq 0,c=\frac{1}{2}$$
  • $$a=\displaystyle \frac{3}{2},b\neq 0,c=-\dfrac{1}{2}$$
The graph of the function $$y = f (x)$$ has a unique tangent at the point $$(e^{a} ,0)$$ through which the graph passes then $$\displaystyle \lim_{x\rightarrow e^{a}}\frac{log_{e}\{1+7f(x)\}-sinf(x)}{3f(x)}$$ is
  • $$1$$
  • $$2$$
  • $$0$$
  • $$-1$$
Assertion(A): $$f(x)=\left\{\begin{array}{ll}x^{2}\sin(\frac{1}{x}) , & x\neq 0\\0, & x=0\end{array}\right.$$ is continuous at $${x}=0$$
Reason(R): Both $$h(x)=x^{2},g(x)=
\left\{\begin{array}{ll}\sin(\frac{1}{x}) , & x\neq 0\\0, & x=0\end{array}\right.$$are continuous at $$x = 0$$
  • Both A and R are true and R is the correct explanation of A
  • Both A and R are true and R is not the correct explanation of A
  • A is true but R is false
  • R is true but A is false
Let $$a, b, c \epsilon R^+$$ and $$\displaystyle\lim_{n\rightarrow \infty }\sum_{k=1}^{n}\displaystyle \frac{n}{(k+an)(k+bn)}=\displaystyle \frac{A}{a-b}ln\displaystyle \frac{a(b+B)}{b(a+C)}, a \neq b,$$ then $$(A + B + C)$$ is equal to
  • 2
  • 3
  • 4
  • 5
If f (x)$$=\left \{ \displaystyle \frac{|x+2|}{tan^{-1}(_{2}x+2)} \right.x\neq -2$$
$$x=-2,$$ then f(x) is
  • continuous at $$x = - 2$$
  • not continuous at $$x = - 2$$
  • differentiable at $$x = - 2$$
  • continuous but not differentiable at $$ x = - 2$$
The function $$f(x) = x - |x - x^2|, -1 \leq x \leq 1$$ is continuous on the interval
  • $$[-1, 1]$$
  • $$[-1, 2]$$
  • $$[-1, 1]- \{0\}$$
  • $$(-1, 1)- \{0\}$$
$$\displaystyle \lim_{x\to\infty}{\displaystyle \frac{2\sqrt{x}+3\sqrt [\Large 3]{x}+4\sqrt [\Large 4]{x}+...+n\sqrt [\Large n]{x}}{\sqrt{(2x-3)}+\sqrt[\Large 3]{(2x-3)}+\sqrt[\Large 4]{(2x-3)}+...+\sqrt[\Large n]{(2x-3)}}}$$ is equal to
  • $$1$$
  • $$\infty$$
  • $$\sqrt{2}$$
  • None of these
If $$f(x) \displaystyle =\frac{3x^2 + ax + a + 1}{x^2 + x - 2}$$, then which of the following can be correct?
  • $$\displaystyle \lim_{x \rightarrow 1} f(x)$$ exists $$\Rightarrow a = - 2$$
  • $$\displaystyle \lim_{x \rightarrow -2} f(x)$$ exists $$\Rightarrow a = 13$$
  • $$\displaystyle \lim_{x \rightarrow 1} f(x) = \dfrac{4}{3}$$
  • $$\displaystyle \lim_{x \rightarrow -2} f(x) = -\dfrac{1}{3}$$
If the function $$f(x)= \left\{\begin{matrix}
(1+\left | \tan x \right |)^{ \displaystyle \frac{p}{\left| \tan {x} \right|}} &, -\frac{\pi }{3}< x< 0 \\ \\
 q& x=0\\ \\
 e^{ \displaystyle \frac{\sin {3x}}{\sin {2x}}},& 0\: < \, x\, < \frac{\pi }{3}
\end{matrix}\right.$$
is continuous at $$x=0$$, then 

  • $$\displaystyle \mathrm{p}=\frac{3}{2}$$
  • $$\displaystyle \mathrm{p}=\frac{2}{3}$$
  • $$\log_{e}\mathrm{q}=\mathrm{p}$$
  • $$\mathrm{q}=2$$
Let $$\tan \alpha .x+\sin \alpha .y=\alpha $$ and $$\alpha \ \text{cosec} \alpha .x+\cos \alpha .y=1$$ be two variable straight line, $$\alpha $$ being the parameter. Let $$P$$ be the point of intersection of the lines. In the limiting position when $$\alpha \rightarrow 0$$, the point $$P$$ lies on the line
  • $$x=2$$
  • $$x=-1$$
  • $$y+1=0$$
  • $$y=2$$
if $$f\left( x \right) = \left\{ {\matrix{   {\cos \left[ x \right],} & {x \ge 0}  \cr    {\left| x \right| + a,} & {x < 0}  \cr 
 } } \right\}$$ Find
the value of a , given that $$\mathop {\lim }\limits_{x \to 0} f\left( x \right)$$  exists,
where[.]  denotes
  • -1
  • 2
  • 1
  • 0
$$f(x) = \left\{\begin{matrix}(3/x^{2})\sin 2x^{2} & if x M 0 \\\dfrac {x^{2} + 2x + c}{1 - 3x^{2}}  & if\ x \geq 0, x \neq \dfrac {1}{\sqrt {3}}\\ 0 & x = 1/ \sqrt {3}\end{matrix}\right.$$ then in order that $$f$$ be continuous at $$x = 0$$, the value of $$c$$ is
  • $$2$$
  • $$4$$
  • $$6$$
  • $$8$$
$$\displaystyle\underset{x\rightarrow 0}{Lt}\left(cosec x-\dfrac{1}{x}\right)=?$$
  • $$0$$
  • $$1/2$$
  • $$1$$
  • Does not exits
For each $$t\in R$$, let [t] be the greatest integer less than or equal to t. Then, 
$$\underset { { x\rightarrow 0 }^{ + } }{ lim } x([\frac { 1 }{ x } ]+[\frac { 2 }{ x } ]+...+[\frac { 15 }{ x } ])$$
  • is equal to 0
  • is equal to 15
  • is equal to 120
  • does not exist (in R)
If $$\phi (x) =\displaystyle \lim_{n \rightarrow \infty} \frac{x^{2n} f(x) + g(x)}{1 + x^{2n}}$$, then
  • $$\phi (x) = g(x)$$ for all x $$\in$$ R
  • $$\phi (x) = f(x)$$ for all x $$\in$$ R
  • $$\left\{\begin{matrix}g(x) & for -1 < x < 1\\ f(x) & for |x| \geq 1\end{matrix}\right.$$
  • $$\left\{\begin{matrix}g(x) & for |x| < 1\\ f(x) & for |x| > 1 \\\displaystyle \frac{f(x) + g(x)}{2} & for |x| = 1\end{matrix}\right.$$
If $$\displaystyle f(x) = \frac{x - e^x + cos  2x}{x^2}, x \neq 0$$, is continuous at $$x = 0$$, 
where [x] and {x} denotes the greatest integer and fractional part functions, respectively.

Then which of the following is correct?
  • $$f(0) = 5/2$$
  • $$[f(0)] = - 2$$
  • $$\{ f(0) \} = - 0.5$$
  • $$[f(0)]\{ f(0) \} = - 1.5$$
Let $$f(x) \displaystyle = \frac{x^2 - 9x + 20}{x -[x]}$$ where [x] is the greatest integer not greater than $$x$$, then
  • $$\displaystyle \lim_{x \rightarrow 5^-} f(x) = 0$$
  • $$\displaystyle \lim_{x \rightarrow 5^+} f(x) = 1$$
  • $$\displaystyle \lim_{x \rightarrow 5} f(x)$$ does not exists
  • $$none\ of\ these$$
If $${ x }_{ 1 },{ x }_{ 2 },{ x }_{ 3 },..,{ x }_{ n }$$ are the roots of the equation $$x^n+ax+b=0,$$ then the value of $$\left( { x }_{ 1 }-{ x }_{ 2 } \right) \left( { x }_{ 1 }-{ x }_{ 3 } \right) \left( { x }_{ 1 }-{ x }_{ 4 } \right) ...\left( { x }_{ 1 }-{ x }_{ n } \right) $$ is equal to
  • $$n{ { x }_{ 1 } }^{ n-1 }+a$$
  • $$n{ { x }_{ 1 } }^{ n-1 }$$
  • $$nx-1+b$$
  • $$n{ { x }_{ 1 } }^{ n-1 }+b$$
STATEMENT-1 : $$\displaystyle \lim_{x \rightarrow 0} [x] \left \{ \frac{e^{1/x} - 1}{e^{1/x} + 1} \right \}$$ (where [.] represents the greatest integer function) does not exist.
STATEMENT-2 : $$\displaystyle \lim_{x \rightarrow 0} \left ( \frac{e^{1/x} - 1}{e^{1/x} + 1} \right )$$ does not exists.
  • STATEMENT-1 is True, STATEMENT-2 is True; STATEMENT-2 is a correct explanation for STATEMENT-1
  • STATEMENT-1 is True, STATEMENT-2 is True; STATEMENT-2 is NOT a correct explanation for STATEMENT-1
  • STATEMENT-1 is True, STATEMENT-2 is False
  • STATEMENT-1 is False, STATEMENT-2 is True
If $$\displaystyle\lim_{x\rightarrow a}{(f(x)+g(x))}=2$$ and $$\displaystyle\lim_{x\rightarrow a}{(f(x)-g(x))}=1$$, 
then the value of $$\displaystyle\lim_{x\rightarrow a}{f(x)g(x)}$$ is?
  • Does not exist
  • Exists and is $$\displaystyle\frac{3}{4}$$
  • Exists and is $$\displaystyle-\frac{3}{4}$$
  • Exists and is $$\displaystyle\frac{4}{3}$$
$$x$$$$1$$$$2$$$$3$$$$4$$$$5$$
$$f(x)$$$$4$$$$3$$$$7$$$$1$$$$3$$
The function f is continuous on the closed interval $$[1, 5]$$ and values of the function are shown in the table above. If the values in the table are used to calculate a trapezoidal sum, the approximate value of $$\int_{1}^{5}f(x)dx$$ is
  • $$14$$
  • $$14.5$$
  • $$15$$
  • $$29$$
$$\quad \lim _{ n\rightarrow \infty  }{ \cfrac { 1 }{ n } \sum _{ r=1 }^{ 2n }{ \cfrac { r }{ \sqrt { { n }^{ 2 }+{ r }^{ 2 } }  }  }  } $$ equal to:
  • $$1+\sqrt { 5 } $$
  • $$-1+\sqrt { 5 } $$
  • $$1+\sqrt { 2 } $$
  • $$1+\sqrt { 2 } $$
The value of $$\lim _{ x\rightarrow 0 }{ \left( { \left( \sin { x }  \right)  }^{ 1/x }+{ \left( 1+x \right)  }^{ \sin { x }  } \right)  } $$ whre $$x> 0$$ is
  • $$0$$
  • $$-1$$
  • $$1$$
  • 2
0:0:1


Answered Not Answered Not Visited Correct : 0 Incorrect : 0

Practice Class 11 Commerce Applied Mathematics Quiz Questions and Answers